9

Short version of the problem: Given 8 +Q charges and 8 -Q charges in 3D, can I find an arrangement in which their potential has its leading non-zero term proportional to $1/r^5$?

Step by step description and motivation?

  1. Single +Q charge: In physics the potential of a single charge is $\propto Q/r$. A single charge +Q therefore has a potential whose leading term in the $1/r$ expansion is $1/r$.

  2. +Q and -Q: For two charges, one positive and one negative, of the same magnitude, the potential they produce has no $1/r$ component and its leading term is $1/r^2$. The arrangement that achieves this is a simple line where the charges sit at $(a,0,0)$ and $(-a,0,0)$.

  3. 2x +Q and 2x -Q: Continuing with the pattern, we can take the previous configuration of 2 charges, multiply them by -1, and move it by $2a$ in the so far unused orthogonal direction. After centering, we'd end up with a square whose vertices contain the charges, with $(-a,-a,0)$ and $(a,a,0)$ being +Q and $(-a,a,0)$ and $(a,-a,0)$ being -Q. Such a configuration has a potential whose leading term is $1/r^3$.

  4. 4x +Q and 4x -Q: Equivalently, we can build a cube with alternating charge signs in its vertices and the resulting potential is $\propto 1/r^4$.

The question now is: with 8 +Q charges, and 8 -Q, can I find a configuration that follows this trend, i.e. whose potential has a leading term of $1/r^5$? The procedure for generating the highly symmetric configuration that I used in the steps 1. - 4. relied on having new orthogonal dimensions in which to shift the copied and sign-flipped version of the previous configuration. Such a move can't be used anymore for $1/r^5$ in the 3D space since we already ran out of dimensions.

Could you find such a configuration and if not, could you prove that the charges can't be arranged in such a way in 3D?

Here's a visualization of the approach: Arranging 2^N charges in 3D

SSF
  • 1,372
  • The obvious configuration to try would be two of your 1/r^4 configurations side by side but as opposite charge. Have you considered that? – Simon Goater Mar 27 '25 at 11:48
  • 2
    I tried that as an answer and was downvoted so fast I killed it. Now I'm thinking that while the average over angular coordinates would drop off like $1/r^5$, the pointwise field might still have $1/r^4$ terms. – Oscar Lanzi Mar 27 '25 at 13:16
  • A potential duplicate of https://physics.stackexchange.com/questions/65651/hexadecapole-potential-using-point-particles – Davide Munari Mar 27 '25 at 20:30
  • @OscarLanzi do you want to write up your attempt anyway? Because the physics site tried your answer without a formal reasoning https://physics.stackexchange.com/a/66267/290063 – Benjamin Wang Mar 27 '25 at 21:13
  • @BenjaminWang Since then I have identified an arrangement that works more definitively. My new answer describes this. – Oscar Lanzi Mar 28 '25 at 00:34

2 Answers2

8

It can be done, but not with the originally proposed method of simply translating a cube (which was tried as an answer and quickly downvoted, then killed). It turns out that with spatial dimensions limited to three this does not completely eliminate the $1/r^4$ terms associated with a single cube. We need a more subtle approach, and maybe a little number theory on the side.

Imagine four positive charges arranged at the vertices of a regular tetrahedron. Of course the field decreases only as $1/r$ as $r\to\infty$, but note that the next term is proportional to $1/r^4$. The square and cube terms cancel out because the tetrahedral arrangement gives no dipole or quadrupole moment.

We should be more precise about the second term. In addition to $1/r^4$ there is a factor from the edge length $a$, which comes from the octupole moment. This factor is proportional to $a^3$, so we have an overall proportionality to $a^3/r^4$.

Now suppose we have four of these tetrahedra:

  • The original tetrahedron of positive charges with edge length $a$

  • The next one with negative charges, having edge length $9a$. This tetrahedron is concentric with the original with its vertices oriented the same way. The latter properties apply also to the remaining tetrahedra.

  • The third tetrahedron has negative charges and edge length $10a$, with the concentric and orientation properties noted above.

  • The last tetrahedron is positively charged with edge length $12a$.

Why the ratio $1:9:10:12$? Well, we noted above that the octupole moment contribution to the potential field is proportional to the cube of the edge length. And the cubes for the positive charges ($1^3+12^3$) are balanced by the ones for the negative ones ($9^3+10^3$): the famous Ramanujan taxicab number of $1729$! We could also have used other cubic taxicab numbers such as $4104=2^3+16^3=9^3+15^3$, in which case the edge-length ratios would be $2:9:15:16$. With this balancing the octupole moments as well as the charges cancel, and the next term was verified numerically as proportional to $1/r^5$ (or including the edge length, $a^4/r^5$).

The constant of this proportionality depends on angular position, as it must in order to satisfy the Laplace equation with any exponent other than $0$ or $1$.

Oscar Lanzi
  • 48,208
4

You don't need $8$ pairs of $\pm$ charges. $4$ pairs of $\pm$ charges is enough to create a $\frac1{r^5}$ potential (see approach 1).

If you really want to use $8$ pairs of $\pm$ charges, see approach $2$ (based on ideas metioned in question).


Approach 1 - placing charges on unit circle on a plane.

For integer $n > 0$ and $k = 0,\ldots,2n-1$, let $\alpha_k = \frac{k\pi}{n}$ and $q_k = (\cos \alpha_k,\sin \alpha_k,0)$.

Place a charge $(-1)^k$ at each $q_k$. At point $p = (x,y,z) = (r\sin\theta\cos\varphi,r\sin\theta\sin\varphi,r\cos\theta)$ where $r > 1$, the potential of these charges equals to

$$\Phi = \sum_{k=0}^{2n-1} \frac{(-1)^k}{|p - q_k|} = \sum_{k=0}^{2n-1}(-1)^k\sum_{\ell=0}^\infty\frac{|q|^{\ell}}{|p|^{\ell+1}}P_\ell(\hat{q}_k\cdot\hat{p}) = \sum_{\ell=0}^\infty \frac{1}{r^{\ell+1}} f_\ell(\hat{p})$$ where $P_\ell$ are Legendre polynomials, $\hat{p}, \hat{q}_k$ are unit vectors in directions of $p$, $q_k$ and $$f_\ell(\hat{p}) = \sum_{k=0}^{2n-1} (-1)^k P_\ell(\hat{q}_k\cdot\hat{p})$$ In order for these to be a $\frac1{r^{n+1}}$ potential, we need $f_0,\cdots,f_{n-1}$ identically zero while $f_n$ doesn't. Let $$g_\ell(\hat{p}) = \sum_{k=0}^{2n-1}(-1)^k (\hat{q}_k\cdot\hat{p})^\ell =\sin^\ell\theta \sum_{k=0}^{2n-1}(-1)^k\cos^\ell(\alpha_k-\varphi) $$ Since for each $m$, $P_0(t),P_1(t),\ldots P_m(t)$ form a basis for polynomials in $t$ with degree $\le m$.
This condition on $f_0,\ldots,f_n$ is equivalent to $g_0,\ldots,g_{n-1}$ identically zero but $g_n$ doesn't.

For $\ell < $n,

$$\require{cancel} \begin{align} \sum_{k=0}^{2n-1}(-1)^k\cos^\ell(\alpha_k-\varphi) &= \sum_{k=0}^{2n-1}\sum_{s=0}^{\ell}\frac{(-1)^k}{2^\ell}\binom{\ell}{s}e^{i(\ell-2s)(\alpha_k - \varphi)}\\ &= \sum_{s=0}^{\ell}\frac1{2^\ell}\binom{\ell}{s}e^{-i(\ell-2s)\varphi}\sum_{k=0}^{2n-1}(-e^{i(\ell-2s)\alpha_1})^k\\ &= \sum_{s=0}^{\ell}\frac1{2^\ell}\binom{\ell}{s}e^{-i(\ell-2s)\varphi} \color{red}{\cancelto{0}{\color{gray}{\frac{1-e^{i(\ell-2s)(2n\alpha_1)}}{1 + e^{i(\ell-2s)\alpha_1}}}}} \\ &= 0 \end{align} $$ So $g_0,\ldots,g_{n-1}$ do vanish identically. For a similar expansion in $s$ for $\ell = n$, most terms cancel like above. There are only two exceptions, $s = 0$ or $s = n {}^{\color{blue}{[1]}}$. The end result is

$$g_n = \frac{4n}{2^n}\sin^n(\theta)\cos(n\varphi)$$

doesn't vanish identically.

As a result, the potential from placing $n$ pairs of $\pm$ charges on vertices of regular $2n$-gon on unit circle in $xy$-plane is a $\frac{1}{r^{n+1}}$ potential:

$$\Phi = A_n \frac{\sin^n(\theta)\cos(n\varphi)}{r^{n+1}} + O(1/{r^{n+2}})$$

where $A_n = \underbrace{\frac{1}{2^n}\binom{2n}{n}}_{\color{blue}{[2]}}\frac{4n}{2^n} = \frac{n}{4^{n-1}}\binom{2n}{n}$

For our case $n = 4$, $4$ pairs of $\pm$ charges on vertices of regular octagon gives us a potential with leading behavior

$$\frac{35}{8}\frac{\sin^4(\theta)\cos(4\varphi)}{r^5}$$

notes

  • $\color{blue}{[1]}$ - when $\ell = n$ and $s = 0$ or $n$, $$e^{i(\ell-2s)\alpha_1} = e^{\pm\pi i} = -1 \implies \sum\limits_{k=0}^{2n-1}(-e^{i(\ell-2s)\alpha_1})^k = \sum\limits_{k=0}^{2n-1} 1 = 2n \ne 0$$
  • $\color{blue}{[2]}$ - coefficient of $x^n$ in $P_n(x)$.

Approach 2 - placing charges on projected vertices of a hypercube.

For an $n > 0$, let $V = \{ -1, 1 \}^n$ be the set of vertices of hypercube $[-1,1]^n$. Let $\eta_1,\ldots,\eta_n$ be any $n$ non-zero vectors in $\mathbb{R}^3$. For any $v = (\epsilon_1,\ldots,\epsilon_n) \in V$, let

$$q_v = \sum_{k=1}^n \epsilon_k \eta_k\quad\text{ and }\quad \epsilon_v = \prod_{k=1}^n \epsilon_k$$

By placing a charge $\epsilon_v$ on each $q_v$, we obtain a configuration of $2^{n-1}$ pairs of $\pm$ charges. Let $\Phi$ be its potential. Let $R = \sum\limits_{k=1}^n |\eta_k|$, it is clear all $|q_v| \le R$. At point $p$ where $r = |p| > R$, we can expand $\Phi$ like what we have done before:

$$\Phi = \sum_{v\in V}\frac{\epsilon_v}{|p - q_v|} = \sum_{\ell = 0}^\infty \frac{f_\ell(\hat{p})}{r^{\ell+1}} \quad\text{ where }\quad f_\ell(\hat{p}) = \sum_{v\in V} \epsilon_v \underbrace{P_\ell(\hat{q}_v\cdot \hat{p}) |q_v|^\ell}_{*1}$$

Expand $q_v$ in $(*1)$ in terms of $\epsilon_k$, it is not hard to see $(*1)$ is a polynomial in $\epsilon_k$ with degree at most $\ell$.

Since $\epsilon_v = \prod_{k=1}^n \epsilon_k$, if $\ell < n$, $(*1)$ doesn't have enough $\epsilon_*$ to make all $\epsilon_k$ have even power. As a result, their sum over $V$ vanishes. This means $f_0, \cdots, f_{n-1}$ vanish identically.

If one do the same thing to $f_n(\hat{p})$ and carry out the summation over $\epsilon_*$, one obtain a horrible mess. I'm not going to write down the actual mess. The only thing we need is following:

If one let $\tau_k = \eta_k\cdot \hat{r}$, then $f_n(\hat{p})$ is a polynomial in $\tau_1,\ldots,\tau_k$ with degree $n$ and has the form: $$f_n(\hat{p}) = 2^n n! (c_0 \prod_{k=1}^n \tau_k + O(\tau_*^{n-2}))$$ where $c_0$ is the coefficient of $t^n$ in $P_n(t)$.

Choose a coordinate system where none of the $\eta_k$ is parallel to $z$-axis and set $\hat{p} = (\cos\phi,\sin\phi,0)$ to be points on unit circle of $xy$-plane. For any $\eta_k = (x_k,y_k,z_k)$, we have

$$\eta_k \cdot \hat{p} = x_k \cos\phi + y_k \sin\phi = \frac{x_k-iy_k}{2} e^{i\phi} + \frac{x_k+y_ki}{2}e^{-i\phi}$$ Substitute these back into $f_n(\hat{p})$, one find on the unit circle, $f_n(\hat{p})$ is a linear combination $e^{in\phi},e^{i(n-1)\phi},\cdots,e^{-in\phi}$.

If $f_n(\hat{p})$ is identically zero, then it need to vanish on the unit circle. This in turn require all coefficients of $e^{im\phi}$ to vanish. However, the coefficient for $e^{in\phi}$ is $n!c_0\prod_{k=1}^n(x_k - y_ki) \ne 0$, this means $f_n(\hat{p})$ doesn't vanish identically and hence $\Phi$ is a $\frac{1}{r^{n+1}}$ potential.

As a concrete example, set $n = 4$ and all $\eta_k$ to $(0,0,1)$, following configuration of charges: $$\begin{align} & +1 \text{ charges at } (0,0,2), (0,0,-2),\\ & -4 \text{ charges at } (0,0,1), (0,0,-1)\\ \text{ and } & +6 \text{ charge at } (0,0,0) \end{align}$$ give us a $\frac1{r^5}$ potential.

achille hui
  • 125,323
  • However $2n\alpha_1 = 2\pi$, regardless of $\ell$ and $s$, so it's not really clear why some terms make an exception when $\ell = n, s = 0, ...$. I mean the final result is good, so I think it's only a problem of a step justification. – Davide Munari Mar 28 '25 at 15:01
  • @quinzio when $\ell = n$, $s = 0$ or $n$, $e^{i(\ell-2s)\alpha_1} = e^{\pm i\pi} = -1$. The denominator in the grayed expression also becomes $0$. The sum over $k$ of $(-e^{i(\ell-2s)\alpha_i})^k$ becomes a sum of $1$s. – achille hui Mar 28 '25 at 15:17
  • That's right. Sorry for the annoyance, Achille. – Davide Munari Mar 28 '25 at 15:23